LSAT 91 – Section 2 – Question 21

You need a full course to see this video. Enroll now and get started in less than a minute.

Target time: 1:24

This is question data from the 7Sage LSAT Scorer. You can score your LSATs, track your results, and analyze your performance with pretty charts and vital statistics - all with a Free Account ← sign up in less than 10 seconds

Question
QuickView
Type Tags Answer
Choices
Curve Question
Difficulty
Psg/Game/S
Difficulty
Explanation
PT91 S2 Q21
+LR
Necessary assumption +NA
A
6%
153
B
6%
155
C
22%
156
D
5%
155
E
61%
163
147
156
166
+Harder 145.724 +SubsectionMedium

This is an NA question.

The stimulus starts with OPA. Many scientists hypothesize that there’s a “light-absorbing medium” because the existence of a light-absorbing medium would explain why other star systems are only dimly visible from Earth. With the word “but,” the author signals the transition from context to argument. The author opens with the conclusion that there’s no reason to believe that the light-absorbing medium hypothesis is correct. Okay, why? Because the low visibility of other star systems is already fully explained by the general theory of relativity.

What assumption is required? If you already see it, then you can go into hunt mode but I’ll proceed from here as if you don’t and use POE.

Answer Choice (A) says that the low visibility of other star systems wouldn’t be adequately explained by the existence of a light-absorbing medium. So basically (A) contradicts other people’s premise. OPP said that the medium’s existence would explain but (A) says it wouldn’t. This isn’t necessary. In general, if you want to weaken someone else’s argument, you don’t have to contradict their premises. You could, but it’s not a requirement.

Answer Choice (B) says that light-absorbing medium hypothesis requires it to adequately explain the low visibility of other star systems. This is also unnecessary. (B) seems to think that the argument is different from what it actually is. If the argument had been the following, then (B) would be right.

Premise: light-absorbing medium hypothesis fails to adequately explain the low visibility of other star systems

Conclusion: light-absorbing medium hypothesis is false

This is the argument that (B) has in mind. But the actual argument is nothing like this. The premise is different and so is the conclusion. The conclusion in the actual argument isn’t that the hypothesis is false; rather, it’s just that there’s no reason to believe that it’s true. This is a distinction that Flaw questions repeatedly test. The author is concluding that OPA failed to give good reasons for the hypothesis, not that the hypothesis is false. The premise is also different. The actual premise is that the phenomenon of low visibility is already explained by some other theory, not that the OPA hypothesis fails to adequately explain the phenomenon.

Answer Choice (C) is similar to (B). It seems to think that the argument is something else. (C) says that if there’s some phenomenon that a hypothesis adequately accounts for and that is not adequately accounted for by an existing theory, then that hypothesis is likely to be correct. If the argument had been the following, then (C) would be right.

Premise: low visibility of other star systems is a phenomenon that existing theory can’t adequately account for but is adequately accounted for by the light-absorbing medium hypothesis

Conclusion: light-absorbing medium hypothesis is likely correct

The actual premise in the argument is just the opposite. It’s saying that an existing theory does adequately account for the phenomenon. The actual conclusion is also pushing in the opposite direction.

Answer Choice (D) says that most proponents of the light-absorbing medium hypothesis accept the general theory of relativity. That’s not necessary. It’s fine for the argument if OPA rejects the general theory of relativity. The argument is just as strong as it ever was.

Correct Answer Choice (E) says that the general theory of relativity does not depend on the light-absorbing medium hypothesis. Yeah, that must be true. If the general theory did depend on the hypothesis, then the argument would fall apart. There would be reason to believe that the hypothesis is correct.

Take PrepTest

Review Results

Leave a Reply